roberts review -release version - final - 050211pednephboardreview.com/welcome_files/roberts...

25
Robert’s Review of Pediatric Nephrology Robert Samuel Gillespie, MD, MPH KidneyWeb Media Arlington, Texas KidneyWeb.net

Upload: others

Post on 21-Aug-2020

6 views

Category:

Documents


1 download

TRANSCRIPT

Page 1: Roberts Review -Release version - FINAL - 050211pednephboardreview.com/Welcome_files/Roberts Review... · I. Fetal and Neonatal Nephrology 1 II. Fluid, Electrolyte and Acid-Base Balance

Robert’s Review of

Pediatric Nephrology

Robert Samuel Gillespie, MD, MPH

KidneyWeb Media Arlington, Texas

KidneyWeb.net

Page 2: Roberts Review -Release version - FINAL - 050211pednephboardreview.com/Welcome_files/Roberts Review... · I. Fetal and Neonatal Nephrology 1 II. Fluid, Electrolyte and Acid-Base Balance

Introduction

vii

Contents in Brief

Introduction iii Contents ix

I. Fetal and Neonatal Nephrology 1

II. Fluid, Electrolyte and Acid-Base Balance 17

III. Diagnostic Techniques 43

IV. Glomerular Diseases 59

V. Acute Kidney Injury 123

VI. Cystic, Genetic and Tubular Diseases 137

VII. Urologic Issues 177

VIII. Blood Pressure 209

IX. Chronic Kidney Disease 229

X. Renal Replacement Therapy 247

XI. Renal Transplantation 277

XII. Basic Science and Laboratory Research 309

XIII. Clinical Research, Biostatistics and Education 329 Appendices 369 Index 375

Page 3: Roberts Review -Release version - FINAL - 050211pednephboardreview.com/Welcome_files/Roberts Review... · I. Fetal and Neonatal Nephrology 1 II. Fluid, Electrolyte and Acid-Base Balance

ix

Contents

Introduction ....................................................................................................... iii

Contents ............................................................................................................. ix

I. Prenatal and Neonatal Nephrology ...............................................................1 1. Renal/urinary tract development ..................................................................................................................................1 2. Renal circulation...........................................................................................................................................................2 3. Glomerular filtration.....................................................................................................................................................2 4. Sodium handling...........................................................................................................................................................3 5. Potassium handling.......................................................................................................................................................4 6. Calcium, phosphorus and magnesium handling ...........................................................................................................4 7. Organic molecule handling...........................................................................................................................................6 8. Energy and metabolism ................................................................................................................................................6 9. Urine concentration and dilution ..................................................................................................................................7 10. Renal acid excretion .....................................................................................................................................................7 11. Prenatal renal hormones ...............................................................................................................................................8 12. Renal artery and vein thrombosis in neonates ..............................................................................................................9 13. Edema and ascites.........................................................................................................................................................9 14. Neonatal hypertension ................................................................................................................................................10 15. Voiding function.........................................................................................................................................................12 16. Urinary tract infection ................................................................................................................................................13 17. Hematuria ...................................................................................................................................................................13 18. Renal function.............................................................................................................................................................13 19. Effects of drugs...........................................................................................................................................................14

II. Fluid, Electrolytes and Acid-Base Physiology ...........................................19 Part A: Intravenous fluid therapy .......................................................................................................................................19 20. Maintenance fluids .....................................................................................................................................................19 21. Replacement fluids .....................................................................................................................................................20 Part B: Volume depletion ...................................................................................................................................................20 22. Review of osmolarity, osmolality and tonicity [supplemental]..................................................................................20 23. Isotonic dehydration ...................................................................................................................................................21 24. Hypertonic dehydration ..............................................................................................................................................22 25. Hypotonic dehydration ...............................................................................................................................................22 26. Oral rehydration therapy.............................................................................................................................................23 Part C: Electrolyte disturbances..........................................................................................................................................24 27. Hyponatremia .............................................................................................................................................................24 28. Hypernatremia ............................................................................................................................................................25 29. Hypokalemia...............................................................................................................................................................27 30. Hyperkalemia..............................................................................................................................................................27 31. Hypocalcemia .............................................................................................................................................................29 32. Hypercalcemia ............................................................................................................................................................30 33. Hypo-osmolar syndromes...........................................................................................................................................31 34. Hyperosmolar syndromes ...........................................................................................................................................31 35. Hypomagnesemia .......................................................................................................................................................32 36. Hypermagnesemia ......................................................................................................................................................33 37. Hyperporcelainemia....................................................................................................................................................33 38. Hypophosphatemia .....................................................................................................................................................34 39. Hyperphosphatemia ....................................................................................................................................................35 Part D: Acid-base disturbances...........................................................................................................................................35 40. Metabolic acidosis ......................................................................................................................................................35 41. Metabolic alkalosis .....................................................................................................................................................37 42. Respiratory acidosis....................................................................................................................................................39 43. Respiratory alkalosis...................................................................................................................................................40

Page 4: Roberts Review -Release version - FINAL - 050211pednephboardreview.com/Welcome_files/Roberts Review... · I. Fetal and Neonatal Nephrology 1 II. Fluid, Electrolyte and Acid-Base Balance

Robert’s Review of Pediatric Nephrology by Robert Samuel Gillespie

x

44. Simple and compensated disturbances .......................................................................................................................41 Part F: Edema......................................................................................................................................................................42 45. Edema .........................................................................................................................................................................42

III. Diagnostic Techniques.............................................................................. 45 46. Urinalysis....................................................................................................................................................................45 47. Tests of renal structure and function ..........................................................................................................................48 48. Renal biopsy ...............................................................................................................................................................51 49. Evaluation of renal biopsies .......................................................................................................................................52 50. Serologic tests.............................................................................................................................................................55

IV. Glomerular Diseases................................................................................. 61 51. Acute post- infectious glomerulonephritis .................................................................................................................61 52. Introduction to vasculitis-associated renal diseases [supplemental] ..........................................................................64 53. Henoch-Schönlein purpura .........................................................................................................................................64 54. Systemic lupus erythematosus....................................................................................................................................67 55. Anti-phospholipid syndrome ......................................................................................................................................72 56. Diagnostic use of ANCA testing ................................................................................................................................72 57. Wegener’s granulomatosis .........................................................................................................................................73 58. Microscopic polyangiitis ............................................................................................................................................74 59. Unspecified or “renal-limited” pauci-immune glomerulonephritis ............................................................................75 60. Diabetic nephropathy..................................................................................................................................................75 61. Sickle cell disease and sickle cell trait (Hemoglobin S disease) ................................................................................77 62. Mixed connective tissue disease.................................................................................................................................79 63. Anti-glomerular basement membrane disease [supplemental] and Goodpasture’s syndrome/disease ......................80 64. Human immunodeficiency virus-associated nephropathy..........................................................................................81 65. Alport syndrome (Familial nephritis) .........................................................................................................................84 66. Thin basement membrane disease [AD].....................................................................................................................86 67. Nail-patella syndrome (osteo-onychodysplasia) ........................................................................................................88 68. Fabry disease ..............................................................................................................................................................90 69. IgA nephropathy .........................................................................................................................................................92 70. Rapidly progressive glomerulonephritis.....................................................................................................................94 71. Nephritis with low serum complement.......................................................................................................................95 72. Infantile nephrotic syndrome......................................................................................................................................96 73. Minimal change disease..............................................................................................................................................97 74. Membranous nephropathy ........................................................................................................................................101 75. Focal segmental glomerulosclerosis .........................................................................................................................103 76. Mesangial proliferative glomerulonephritis (including IgM and C1q nephropathy) ...............................................108 77. Diffuse proliferative glomerulonephritis ..................................................................................................................109 78. Membranoproliferative glomerulonephritis (MPGN) ..............................................................................................110 79. Secondary nephrotic syndrome [supplemental] .......................................................................................................115 80. Pathophysiology of nephrotic syndrome ..................................................................................................................115 81. Systemic manifestations and complications of nephrotic syndrome........................................................................116 82. Hemolytic uremic syndrome (HUS).........................................................................................................................118

V. Acute Kidney Injury................................................................................ 125 Part A: Types of acute kidney injury ................................................................................................................................125 83. Oliguric and non-oliguric acute kidney injury .........................................................................................................125 84. Volume depletion .....................................................................................................................................................126 85. Acute tubular necrosis ..............................................................................................................................................126 86. Obstructive uropathy ................................................................................................................................................127 87. Renal artery and vein thrombosis .............................................................................................................................128 88. Cortical, medullary and papillary necrosis ...............................................................................................................129 89. Acute interstitial nephritis ........................................................................................................................................130 90. Tumor lysis syndrome ..............................................................................................................................................130 91. Heme pigment nephropathy [supplemental].............................................................................................................131 92. Hepatorenal syndrome ..............................................................................................................................................132 93. Nephrotoxic agents ...................................................................................................................................................134

Page 5: Roberts Review -Release version - FINAL - 050211pednephboardreview.com/Welcome_files/Roberts Review... · I. Fetal and Neonatal Nephrology 1 II. Fluid, Electrolyte and Acid-Base Balance

Contents

xi

Part B: Management of acute kidney injury .....................................................................................................................134 94. Fluid therapy.............................................................................................................................................................134 95. Electrolyte disturbances............................................................................................................................................134 96. Nutrition....................................................................................................................................................................134 97. Drugs ........................................................................................................................................................................135 98. Dialysis/hemofiltration .............................................................................................................................................135

VI. Cystic, Genetic and Tubular Diseases .....................................................139 Part A: Cystic diseases......................................................................................................................................................139 99. Introduction to cystic diseases [supplemental].........................................................................................................139 100. Review of basic Mendelian genetics [supplemental] ...............................................................................................139 101. Autosomal recessive polycystic kidney disease (arpkd) ..........................................................................................140 102. Autosomal dominant polycystic kidney disease (ADPKD) .....................................................................................142 103. Multicystic renal dysplasia .......................................................................................................................................145 104. Glomerulocystic disease ...........................................................................................................................................147 105. Acquired cystic kidney disease.................................................................................................................................149 106. Solitary cysts.............................................................................................................................................................150 107. Nephronophthisis/medullary cystic kidney disease complex...................................................................................151 108. Medullary sponge kidney .........................................................................................................................................153 109. Tuberous sclerosis ....................................................................................................................................................154 110. von Hippel-Lindau disease [AD] [supplemental].....................................................................................................155 111. Williams syndrome [AD] .........................................................................................................................................156 112. Bardet-Biedel syndrome [ar] ....................................................................................................................................157 113. Beckwith-Wiedemann syndrome .............................................................................................................................157 Part B: Tubular diseases ...................................................................................................................................................158 114. Glucosuria.................................................................................................................................................................158 115. Review of vitamin D metabolism [supplemental] ....................................................................................................159 116. Introduction to rickets [supplemental]......................................................................................................................161 117. X-linked and autosomal dominant hypophosphatemic rickets.................................................................................161 118. Hereditary hypophosphatemic rickets with hypercalciuria [ar] ...............................................................................163 119. Tumor-induced osteomalacia ...................................................................................................................................163 120. Adolescent sporadic hypophosphatemia with osteomalacia ....................................................................................163 121. Primary hyperparathyroidism ...................................................................................................................................164 122. Vitamin D-deficient rickets ......................................................................................................................................165 123. Vitamin D-dependent rickets [ar] .............................................................................................................................165 124. Other causes of hypophosphatemia ..........................................................................................................................165 125. Introduction to renal tubular acidosis [supplemental] ..............................................................................................166 126. Distal (Type 1) renal tubular acidosis.......................................................................................................................167 127. Proximal (Type 2) renal tubular acidosis .................................................................................................................168 128. (Type 4) Renal tubular acidosis with hyperkalemia.................................................................................................169 129. Fanconi syndrome and cystinosis .............................................................................................................................170 130. Dent disease [xl] .......................................................................................................................................................171 131. Lowe (oculocerebrorenal) syndrome [xl] [supplemental] ........................................................................................172 132. Diabetes insipidus.....................................................................................................................................................173 133. Syndrome of inappropriate antidiuretic hormone (SIADH) [supplemental]............................................................174 134. Acute and chronic tubulointerstitial nephritis ..........................................................................................................175

VII. Urologic Issues ....................................................................................179 Part A: UTI .......................................................................................................................................................................179 135. Urinary tract infection ..............................................................................................................................................179 136. Vesicoureteral reflux ................................................................................................................................................181 Part B: Trauma..................................................................................................................................................................182 137. Kidney trauma ..........................................................................................................................................................182 138. Ureteral trauma .........................................................................................................................................................183 139. Bladder and urethral trauma .....................................................................................................................................183 140. Vascular trauma........................................................................................................................................................184 141. Diagnostic evaluation, treatment and sequelae of urinary tract trauma ...................................................................184 Part C: Urolithiasis ...........................................................................................................................................................185 142. Cystinuria..................................................................................................................................................................185

Page 6: Roberts Review -Release version - FINAL - 050211pednephboardreview.com/Welcome_files/Roberts Review... · I. Fetal and Neonatal Nephrology 1 II. Fluid, Electrolyte and Acid-Base Balance

Robert’s Review of Pediatric Nephrology by Robert Samuel Gillespie

xii

143. Idiopathic hypercalciuria ..........................................................................................................................................186 144. Calcium oxalate and calcium phosphate nephrolithiasis ..........................................................................................187 145. Hypocitraturia ...........................................................................................................................................................189 146. Uric acid nephrolithiasis ...........................................................................................................................................190 147. Struvite stones...........................................................................................................................................................191 148. Primary hyperoxaluria (Primary oxalosis) [ar].........................................................................................................191 149. Pathogenesis of urinary stones .................................................................................................................................193 150. Urologic management of urinary stones [supplemental]..........................................................................................194 151. Factitious stones .......................................................................................................................................................195 Part D: Other disorders .....................................................................................................................................................195 152. Enuresis ....................................................................................................................................................................195 153. Anomalies of the urinary tract ..................................................................................................................................197 154. Voiding dysfunction (Neurogenic and non-neurogenic) ..........................................................................................199 155. Disorders of the bladder ...........................................................................................................................................201 156. Wilms’ tumor (Nephroblastoma)..............................................................................................................................201 157. Mesoblastic nephroma ..............................................................................................................................................202 158. Angiomyolipoma ......................................................................................................................................................203 159. Urinary tract obstruction...........................................................................................................................................203 160. Compensatory hypertrophy and adaptation ..............................................................................................................205 161. Renal hypoplasia and oligomeganephronia ..............................................................................................................206

VIII. Blood Pressure..................................................................................... 211 162. Normal vs. abnormal blood pressure ........................................................................................................................211 163. Determinants of blood pressure ................................................................................................................................213 164. Evaluation of elevated blood pressure......................................................................................................................214 165. Acute hypertension ...................................................................................................................................................215 166. Sustained hypertension .............................................................................................................................................216 167. Primary (essential) hypertension ..............................................................................................................................221 168. Treatment of hypertension........................................................................................................................................222 169. End organ effects of hypertension ............................................................................................................................227

IX. Chronic Kidney Disease .......................................................................... 231 Part A: General .................................................................................................................................................................231 170. Epidemiology and etiology.......................................................................................................................................231 171. Pathogenesis .............................................................................................................................................................231 Part B: Metabolic disturbances .........................................................................................................................................232 172. Mineral metabolism, vitamin D and parathyroid hormone ......................................................................................232 173. Skeletal .....................................................................................................................................................................234 174. Lipid metabolism ......................................................................................................................................................235 175. Carbohydrate metabolism.........................................................................................................................................236 176. Growth and endocrine disturbances .........................................................................................................................236 177. Protein metabolism and peptide degradation............................................................................................................239 178. Drug metabolism ......................................................................................................................................................239 Part C: Systemic disturbances...........................................................................................................................................240 179. Cardiovascular ..........................................................................................................................................................240 180. Pulmonary.................................................................................................................................................................241 181. Gastrointestinal/hepatic ............................................................................................................................................241 182. Immunologic.............................................................................................................................................................241 183. Neurologic ................................................................................................................................................................243 184. Hematologic system .................................................................................................................................................244

X. Renal Replacement Therapy................................................................... 249 185. Hemodialysis ............................................................................................................................................................249 186. Hemodialysis for hyperammonemia in newborns ....................................................................................................257 187. Continuous renal replacement therapy (CRRT) .......................................................................................................258 188. Peritoneal dialysis.....................................................................................................................................................260 189. Hemoperfusion and poisoning..................................................................................................................................266 190. Drug therapy and clearances.....................................................................................................................................267

Page 7: Roberts Review -Release version - FINAL - 050211pednephboardreview.com/Welcome_files/Roberts Review... · I. Fetal and Neonatal Nephrology 1 II. Fluid, Electrolyte and Acid-Base Balance

Contents

xiii

191. Psychosocial adaptation............................................................................................................................................268 192. Rehabilitation............................................................................................................................................................269 193. Review of medical ethics [supplemental].................................................................................................................270 194. Ethical problems .......................................................................................................................................................271 195. Systemic complications of dialysis...........................................................................................................................272 196. Nephrogenic systemic fibrosis [supplemental].........................................................................................................276

XI. Renal Transplantation ............................................................................279 197. Recipient selection and preparation..........................................................................................................................279 198. Donor selection.........................................................................................................................................................280 199. Ethical considerations...............................................................................................................................................284 200. Access to transplantation ..........................................................................................................................................286 201. Financial issues.........................................................................................................................................................289 202. Histocompatibility ....................................................................................................................................................290 203. Surgical technique ....................................................................................................................................................294 204. Immunosuppression..................................................................................................................................................295 205. Rejection...................................................................................................................................................................298 206. Complications of renal transplantation.....................................................................................................................301 207. Growth and sexual maturation..................................................................................................................................305 208. Rehabilitation and psychosocial adaptation .............................................................................................................306 209. Graft and patient outcome ........................................................................................................................................307

XII. Basic Science and Laboratory Research ..............................................311 Part A: Immunology and immunologic injury..................................................................................................................311 210. Mechanisms of humoral immunity...........................................................................................................................311 211. Mechanisms of cell-mediated immunity ..................................................................................................................312 212. Mediator systems in immune renal disease ..............................................................................................................313 213. HLA associations......................................................................................................................................................314 214. Transplant immunology............................................................................................................................................315 Part B: Mechanisms of disease .........................................................................................................................................316 215. Mechanisms of renal injury ......................................................................................................................................316 216. Mechanisms of progressive renal disease.................................................................................................................317 217. Renal physiology ......................................................................................................................................................319 218. Renal development ...................................................................................................................................................324 219. Molecular and cellular biology.................................................................................................................................325 220. Genetics in renal disease...........................................................................................................................................326 221. Experimental methods, design and analysis .............................................................................................................327

XIII. Clinical Research, Biostatistics and Education ....................................331 Part A: Biostatistics ..........................................................................................................................................................331 222. Types of variables.....................................................................................................................................................331 223. Distribution of data...................................................................................................................................................331 224. Hypothesis testing.....................................................................................................................................................332 225. Statistical tests ..........................................................................................................................................................333 226. Measurement of association .....................................................................................................................................335 227. Regression ................................................................................................................................................................337 228. Diagnostic tests.........................................................................................................................................................339 229. Systematic reviews and meta-analysis .....................................................................................................................341 Part B: Clinical research and epidemiology .....................................................................................................................344 230. Study types ...............................................................................................................................................................344 231. Bias and confounding ...............................................................................................................................................346 232. Causation ..................................................................................................................................................................347 233. Incidence and prevalence..........................................................................................................................................348 234. Screening ..................................................................................................................................................................348 235. Decision analysis ......................................................................................................................................................348 236. Cost-benefit, cost-effectiveness, and outcomes........................................................................................................350 237. Sensitivity analysis ...................................................................................................................................................351 238. Measurement ............................................................................................................................................................351

Page 8: Roberts Review -Release version - FINAL - 050211pednephboardreview.com/Welcome_files/Roberts Review... · I. Fetal and Neonatal Nephrology 1 II. Fluid, Electrolyte and Acid-Base Balance

Robert’s Review of Pediatric Nephrology by Robert Samuel Gillespie

xiv

239. Evaluating internal validity ......................................................................................................................................353 240. Evaluating external validity......................................................................................................................................356 241. Application of information in patient care ...............................................................................................................357 242. Using the medical literature......................................................................................................................................358 Part C: Adult education.....................................................................................................................................................359 243. Educational theory ....................................................................................................................................................359 244. Feedback and evaluation ..........................................................................................................................................361 245. Teaching methods.....................................................................................................................................................362 246. Educational planning ................................................................................................................................................363 Part D: Research ethics .....................................................................................................................................................364 247. Conflicts of interest and commitment ......................................................................................................................364 248. Professionalism and misconduct in research ............................................................................................................364 249. Principles of research with human subjects..............................................................................................................366

Appendices ...................................................................................................... 371 Appendix A: Abbreviations used......................................................................................................................................371 Appendix B: Genes associated with renal diseases ..........................................................................................................374 Appendix C: Key word associations.................................................................................................................................375

Index ............................................................................................................... 377

Page 9: Roberts Review -Release version - FINAL - 050211pednephboardreview.com/Welcome_files/Roberts Review... · I. Fetal and Neonatal Nephrology 1 II. Fluid, Electrolyte and Acid-Base Balance

1

I. Prenatal and Neonatal Nephrology

1. Renal/urinary tract development

Stages and process of development ! Three different nephric systems develop, all from intermediate mesoderm, in succession:

• Pronephros: Nonfunctional, but induces mesonephros; completely regresses by week 5 ! Also in week 5, the metanephric blastema forms, again, from intermediate mesoderm

• Mesonephros: Takes place of pronephros at 5 weeks ! The first kidney - produces small amount of urine from 6-10 weeks ! Induces metanephros, and regresses at 10 weeks ! In females, fully regresses; in males, part of it becomes male genital system

• Metanephros: Highly functioning renal tissue; becomes the final, permanent kidney o The mesonephric duct is connected to the bladder and later becomes the trigone o The metanephros forms when the mesonephric duct sprouts the ureteric bud, and pokes into the

metanephric blastema o When these two ends connect, a continuous, functional renal system is born

! Ureteric bud then begins a series of bifurcations, fanning out and forming collecting system ! Collecting ducts are not considered part of the nephron because they arise from ureteric bud

! Metanephric blastema becomes nephrons, including glomeruli and tubules o This system produces urine at 10 weeks o Control is reciprocal – each end (ureteric bud and metanephric blastema) releases factors that

induce the other to develop and differentiate ! Therefore, in the absence of a ureteric bud, no kidney will form, even if metanephric

mesenchyme is present • Kidneys then migrate cephalad to their usual location

! As they migrate, the arterial supply changes several times: First from the common iliac arteries, then low on the aorta, then intermediate arteries from the aorta, to the final renal artery higher up on the aorta o An accessory renal artery forms if one or more of these intermediate arteries fails to regress o This is very common –at least one accessory artery can be found in about 1 in 4 kidneys

! Renal veins sprout off the subcardinal vein, which becomes inferior vena cava. Accessory veins are much less common

• All nephrons are formed by birth (if born at term) ! Bladder develops separately, from the urogenital sinus (which also forms the rectum.) Thus one can have a

bladder even with complete renal agenesis ! Compensatory renal growth begins prenatally Role of urine in lung development ! Fetal urine is primary source of amniotic fluid after 16-18 weeks ! Absent or poorly functioning kidneys result in oligohydramnios ! Best indicator of urine production in utero is a full bladder Apoptosis in renal development ! Apoptosis is programmed (or regulated) cell death or a decrease in cell number

• Developing kidney has high cell turnover which allows for remodeling as it develops • Apoptosis helps control this as well as prevent uncontrolled growth

! Multiple signals tell cells to “live” or “die;” net effect (balance) determines outcome • Pro-apoptotic: TGF-!1, p53, cell stretching • Anti-apoptotic: IGF-1, catalase, EGF

! Disruption in this system, with excess apoptosis, can result in renal hypoplasia ! Apoptosis is also increased with developmental injury such as obstruction Drugs/toxins and renal development ! Many drugs associated with developmental abnormalities of urinary tract and other systems – mechanisms for

most not well understood • ACE inhibitors, angiotensin receptor blockers • Anticonvulsants, including phenytoin, phenobarbital, valproic acid, carbamazepime, and lamotrigine • Vitamin A compounds, such as retinoin

Page 10: Roberts Review -Release version - FINAL - 050211pednephboardreview.com/Welcome_files/Roberts Review... · I. Fetal and Neonatal Nephrology 1 II. Fluid, Electrolyte and Acid-Base Balance

Section II: Fluid, Electrolyte and Acid-Base Balance

35

! See chapter 27, Hyponatremia

39. Hyperphosphatemia

Causes of hyperphosphatemia ! Excessive intake/absorption

• Previously, some physicians used massive doses of oral sodium phosphate (Fleets Phospho Soda) off-label for pre-procedure bowel cleanout. Reports of severe hypocalcemia and ARF due to renal calcium phosphate precipitation led to the product’s withdrawal from the market

• Excess vitamin D ! Decreased excretion

• Renal failure • Hypoparathyroidism • Vitamin D (inhibits PTH) • Deficiency of fibroblast growth factor 23 (FGF23) protein or loss-of-function mutation of FGF23 gene

! FGF23 protein inhibits renal phosphate reabsorption ! Gain-of-function mutation of FGF23 gene causes hypophosphatemic rickets

o See chapter 117, Hypophosphatemic rickets ! Intracellular release

• Tumor lysis syndrome • Rhabdomyolysis • Severe hemolysis

Evaluation and management of hyperphosphatemia ! Evaluation

• History and physical, including diet history and medications (prescribed, over-the-counter, vitamins and supplements). Further tests depending on clinical situation and findings: ! Serum total and/or ionized calcium ! Vitamin D levels ! Uric acid ! PTH

! Treatment • Normal renal function

! Treat hypocalcemia if severe or symptomatic ! Hydration – kidneys will excrete phosphorus

• Impaired renal function ! Treat hypocalcemia if severe or symptomatic ! Dietary measures ! Phosphate binders ! Dialysis

Part D: Acid-base disturbances

40. Metabolic acidosis

Signs and symptoms of metabolic acidosis ! Tachypnea ! Abdominal pain ! Vomiting ! Neurologic symptoms – often due to underlying disease - lethargy, irritability, seizures, coma ! Renal failure – edema, hypertension, anemia, uremic symptoms, electrolyte disturbances Anion gap and laboratory evaluation of metabolic acidosis Evaluation and management of metabolic acidosis ! Causes of metabolic acidosis

• Increased acid production – DKA, alcohol ingestion, metabolic defect • Loss of bicarbonate – diarrhea, enteric fistula, toluene, carbonic anhydrase inhibitors

Page 11: Roberts Review -Release version - FINAL - 050211pednephboardreview.com/Welcome_files/Roberts Review... · I. Fetal and Neonatal Nephrology 1 II. Fluid, Electrolyte and Acid-Base Balance

Robert’s Review of Pediatric Nephrology by Robert Samuel Gillespie

36

• Decreased acid excretion – RTA, renal failure ! Anion gap: Na+ – (Cl- + HCO3

-) • Normal ~5-16 (varies by lab) • Some authors also include K in calculation: (Na+ + K+) – (Cl- + HCO3-)

! Evaluation • History and physical – especially ask about vomiting, diarrhea, growth, weight loss, family history, possible

ingestions • Venous or arterial blood gas • Electrolytes, creatinine, urinalysis; calculate anion gap • Osmolar gap: Calculated Posm = 2 x [Na] + [glucose]/18 + [BUN]/2.8

! Sodium, glucose and urea normally account for the vast majority of plasma osmolality ! If calculated Posm is significantly lower than measured Posm, there is some osmotic substance in the

bloodstream, such as ethanol, methanol, ketones or mannitol ! In renal failure various organic solutes accumulate and can cause a mild osmolal gap

• Certain combinations of anion/osmolar gap may be clue to diagnosis

Table 40.1: Use of anion and osmolar gaps in diagnosis

Anion gap Osmolar gap Possible cause " Isopropyl alcohol " " Ethanol/methanol, Ethylene/propylene glycol " Salicylates

• Other selected tests based on clinical findings and suspicion (e.g., blood glucose, organic acid levels,

toxicology screens, etc.) ! Treatment

• Treat underlying disorder • For severe acidosis, treat with sodium bicarbonate

Potential complications of treatment of metabolic acidosis ! Hypervolemia from fluid volume ! Hypernatremia ! Hypercarbia in patient with respiratory problems - the HCO3 may increase pCO2, making acidosis worse ! Hypokalemia – base delivery causes K to move into cells; may reveal underlying K deficit. Common in DKA ! Alkalosis

• From excessive bicarbonate administration • In lactic acidosis or DKA, lactate and ketoacids are converted to bicarbonate when underlying disease

corrected Causes of pseudohypoaldosteronism ! Type 1 pseudohypoaldosteronism is further subdivided into multi-organ and renal forms which are completely

different • Multi-organ form [ar]: Decreased function of ENaC (Epithelial Na Channel in Collecting tubule) plus loss-

of-function mutation of MC receptor; salt wasting in kidney, sweat glands, salivary glands and colon • Renal form [AD]: loss-of-function mutation of MC receptor; does not involve ENaC; salt wasting limited to

kidney • Patients have salt wasting, hyponatremia, hyperkalemia, acidosis, high renin and aldosterone level but not

hypertensive; patients with renal form may have lesser degree of findings ! Type 2 pseudohypoaldosteronism (Gordon syndrome): Mutation in WNK1 or 4

• Excess function (excess Cl- reabsorption) of thiazide-sensitive NaCl cotransporter – “the opposite of Gitelman syndrome”

• Patients have acidosis (key feature), hypertension, hyperkalemia, low renin/aldosterone levels ! Types 1 and 2 have very little in common; names incorrectly infer they are related ! Remember that true (not pseudo) hypoaldosteronism is Type IV RTA ! See chapter 166, Sustained hypertension, for much more information and a table on these disorders Factors that increase the anion gap Causes of metabolic acidosis with increased anion gap

Page 12: Roberts Review -Release version - FINAL - 050211pednephboardreview.com/Welcome_files/Roberts Review... · I. Fetal and Neonatal Nephrology 1 II. Fluid, Electrolyte and Acid-Base Balance

Section III: Diagnostic Techniques

51

Intravenous pyelography Renal ultrasound and Doppler studies Voiding cystourethrogram Magnetic resonance imaging and angiogram Computed tomography Renal artery and vein angiogram

Table 47.2: Urinary tract imaging techniques

Study Indications/advantages Limitations/complications IV pyelogram Evaluate for stones

Anatomic localization of kidneys Good visualization of collecting systems and ureters

Requires IV contrast Largely supplanted by CT and other methods today

Ultrasound Good for evaluation of renal anatomy, size, presence of cysts, hydronephrosis or nephrocalcinosis Noninvasive, radiation-free

More operator-dependent Does not evaluate function

Doppler study Evaluation of renal vessels and blood flow Evaluation for renal artery stenosis Requires no contrast or radiation

Low sensitivity Requires long breath hold for best results Difficult in obese patients

VCUG Evaluation for vesicoureteral reflux Excellent visualization of bladder and urethra Has standardized grading scale

Requires urethral catheterization – small risk of bladder or urethral trauma or infection Requires fluoroscopy – significant radiation exposure May induce astonishing degree of parental anxiety

CT Very fast and high resolution Shows other abdominal organs in addition to kidneys Excellent evaluation for stones, calcifications, hemorrhage, trauma, abdominal pain Less invasive option for angiography

Some studies, such as angiography, require IV contrast (note: noncontrast CT better for identifying stones) Metal implants introduce significant artifact Substantial radiation exposure

MRI No radiation Shows other abdominal organs in addition to kidneys Very sensitive for identification of ectopic renal tissue Permits angiography without iodinated contrast

Gadolinium associated with nephrogenic systemic fibrosis in patients with reduced renal function Takes relatively long time to image – may require sedation in uncooperative children Cannot be done in patients with some metal implants Difficult in very obese or claustrophobic patients

Conventional angiography

Gold standard for diagnosing renal artery stenosis Allows simultaneous therapeutic interventions (e.g. angioplasty) Allows for renal vein renin sampling

Very invasive – requires arterial puncture Requires significant IV contrast and radiation exposure Risks include bleeding, infection and injury to vessels

" Fun fact Inulin is a fructose polymer derived from “Jerusalem artichokes” (actually a type of sunflower) and dahlias. It can be used as a sucrose-free sweetener and is popular among natural food advocates as an alternative to more “artificial” sweeteners such as saccharin or aspartame.

Reference aRodriguez-Soriano J, Vallo A, Castillo G. Renal handling of water and sodium in infancy and childhood: a study

using clearance methods during hypotonic saline diuresis. Kidney Int 1981;20:700-704. PMID 7334744.

48. Renal biopsy

Indications, risks and benefits! Indications

• Persistent non-nephrotic, non-orthostatic proteinuria • Unexplained, atypical or unresponsive nephrotic syndrome • Acute glomerulonephritis, if it’s not clearly post-infectious (post-streptococcal) • Unexplained renal failure • Decline in function of renal allograft

! Relative contraindications • Bleeding disorder or use of anticoagulants or platelet inhibitors

Page 13: Roberts Review -Release version - FINAL - 050211pednephboardreview.com/Welcome_files/Roberts Review... · I. Fetal and Neonatal Nephrology 1 II. Fluid, Electrolyte and Acid-Base Balance

Robert’s Review of Pediatric Nephrology by Robert Samuel Gillespie

66

Figure 53.1. Mesangial hypercellularity in a patient with HSP or IgA nephropathy. Courtesy of Thomas Milligan, MD

Figure 53.2 Focal areas of hypercellularity on PAS stain in HSP/IgAN. Courtesy of Thomas Milligan, MD

Figure 53.3. This slide illustrates the diffuse mesangial hypercellularity typical of HSP/IgAN, although it actually is from a patient with HIV. Courtesy of Kevin McBryde, MD

Figure 53.4. Silver stain elegantly defines the diffuse mesangial hypercellularity seen in IgAN/HSP. This biopsy is actually from a patient with HIV, but may represent an immune complex nephritis rather than classic HIVAN. Courtesy of Kevin McBryde, MD

Treatment ! Tylenol for pain ! Virtually all agree that prednisone also helps improve abdominal pain ! Beyond that, treatment is very controversial with many small, usually uncontrolled studies and no really

conclusive data • Many people use steroids for severe arthritis or rash • Some authors avoid NSAIDs due to risk of GI bleeding, while others feel NSAIDs are safe and acceptable • Most use multiple-drug regimens for severe nephritis, such as:

! Prednisone + Azathioprine is probably most well accepted in pediatrics ! Prednisone + Cyclophosphamide + Dipyridamole ± Heparin or Warfarin

Page 14: Roberts Review -Release version - FINAL - 050211pednephboardreview.com/Welcome_files/Roberts Review... · I. Fetal and Neonatal Nephrology 1 II. Fluid, Electrolyte and Acid-Base Balance

Robert’s Review of Pediatric Nephrology by Robert Samuel Gillespie

68

Histopathology and treatment Table 54.2: Classes of lupus nephritis

Class Histologic findings Clinical findings Treatment/Notes

I Minimal

mesangial

Appears normal on LM Few mesangial deposits on IF/EM

Normal urinalysis Excellent renal prognosis; no therapy required for renal disease

II Mesangial

proliferative

Mesangial hypercellularity Mesangial deposits NO other significant deposits or scarring

Microscopic hematuria and/or proteinuria Normal renal function Normal BP

Excellent renal prognosis; no therapy required for renal disease

III Focal

< 50% of glomeruli show GN (on LM) Subendothelial & mesangial deposits

Hematuria, proteinuria, ±hypertension, ±nephrotic syndrome, ±elevated creatinine

Subclasses of Active and/or Chronic; treatment depends on severity and aggressiveness of disease; ranges from none to same as class IV

IV Diffuse

> 50% of glomeruli show GN (on LM) Subendothelial & mesangial deposits

Most severe form; usually has all of the findings listed for class III

Has subclasses of Segmental or Global plus Active and/or Chronic Usually aggressive treatment; see below

V Membranous

Diffuse BM thickening Subepithelial & mesangial deposits; spikes; very similar to membranous nephropathy

Usually nephritic syndrome; often little or no systemic or serologic evidence of SLE

Treatment not well established; often prednisone; some add cyclosphosphamide, azathioprine or mycophenolate; see text below

VI Advanced sclerosing

Global sclerosis of > 90% of glomeruli; no active GN

Elevated creatinine; slow decline in renal function

None; no active disease; too late to reverse

See also Table 49.1 for details on IF and EM findings. ! Other findings that may be present in lupus nephritis (LN) (see illustrations in overleaf)

• Presence/degree of tubulointerstitial nephritis is important prognostic indicator • Vascular disease • Thrombotic microangiopathy • Tubuloreticular structures

! Classic finding in lupus nephritis, although actual composition and clinical significance not known ! Differential: The only other disease associated with these is HIV-associated nephropathy

o That has a very different clinical, serologic and histologic presentation (collapsing FSGS without immune complexes) so it should be easy to distinguish

! Immunofluorescence • IgG is a hallmark of SLE • Usually many if not all other factors too

! “Full-house” nephritis: Presence of IgG, IgM, IgA, C3, C4 and C1q –classic for SLE ! Occurs in ~ # of patients

! Classification system revised in 2004, with particular changes to classes III and IV as described below • Hopefully, this will help in treatment decisions, which are based on largely on activity or presence of

proliferative lesions • How this will affect outcomes is not known yet

! Class III • Further divided into subclasses of Active (proliferative lesions) and/or Chronic (sclerosing/scarring) (see

table) Table 54.3: Subclasses of class II lupus nephritis

Class Description Synonym III (A) Active lesions Focal proliferative LN III (A/C) Active and chronic lesions Focal proliferative and sclerosing LN III (C) Chronic lesions Focal sclerosing LN

! Class IV • Incorporates the Active/Chronic subclasses as in class III, and further subdivided into Segmental and Global

Page 15: Roberts Review -Release version - FINAL - 050211pednephboardreview.com/Welcome_files/Roberts Review... · I. Fetal and Neonatal Nephrology 1 II. Fluid, Electrolyte and Acid-Base Balance

Section IV: Glomerular Diseases

75

!Focal pauci-immune crescentic necrotizing small vessel vasculitis !Identical renal histologic lesion !May involve multiple organ systems including skin, joints, eyes, CNS, GI tract !ANCA-positive (in most cases)

Wegener’s granulomatosis Microscopic polyangiitis !c-ANCA/PR3 positive (in most cases) !Granulomatous disease !Classic upper respiratory findings including subglottic, tracheal or endobronchial stenosis !Pulmonary involvement including nodules or hemorrhage

!p-ANCA/MPO positive (in most cases) !Not granulomatous !No upper respiratory involvement !Pulmonary involvement uncommon and usually limited to alveolar capillaritis

59. Unspecified or “renal-limited” pauci-immune glomerulonephritis

! Pauci-immune GN that does not fit into the other categories above ! Often involves kidneys only without evidence of multisystem disease

• These patients may go on to develop other systemic manifestations in future ! Often p-ANCA positive

• Sometimes called “p-ANCA vasculitis” although this is not a standardized term ! Histology, treatment and outcomes similar to renal disease in WG and MPA, described above

60. Diabetic nephropathy

Clinical manifestations ! Systemic

• Hyperglycemia is cardinal feature (hypoglycemia occurs due to treatment) • Weight loss, dehydration and ketoacidosis can occur in patients with type 1 diabetes • The metabolic syndrome including obesity, hypertension and hyperlipidemia is common in patients with type

2 diabetes • Long-term complications of diabetes mellitus include

! Retinopathy ! Neuropathy ! Nephropathy ! Impaired wound healing, especially foot lesions

! Renal • Acute

! Tubules can reabsorb glucose at a rate of ~250-400 mg/min/1.73 m2 ! More practically speaking, when blood glucose exceeds the tubular reabsorptive threshold (180-200

mg/dl), glucose enters urine and causes osmotic diuresis o Can lead to profound dehydration in patients with diabetic ketoacidosis

! Acidosis in DKA - due to accumulation of ketoacids beta-hydroxybutyrate and acetoacetate o Acetone is the other ketone but it is not an acid

• Long-term ! Microalbuminuria progressing to macroalbuminuria and ultimately renal failure

Histopathology (*Feature found in all specimens) ! Same for type 1 or 2 ! Increased renal size ! Light microscopy

• Diffuse mesangial expansion* ! This often progresses to nodular glomerulosclerosis (Kimmelstiel-Wilson lesions)

• Arteriolar hyalinosis of afferent and efferent arterioles (the latter highly specific for diabetic nephropathy) ! From luminal narrowing all the way to complete obliteration of arterioles

• “Exudative lesions” – subendothelial hyalinosis • Interstitial fibrosis in late stages

! Immunofluorescence • Linear glomerular and tubular basement membrane, and Bowman’s capsule staining for IgG and albumin*

Page 16: Roberts Review -Release version - FINAL - 050211pednephboardreview.com/Welcome_files/Roberts Review... · I. Fetal and Neonatal Nephrology 1 II. Fluid, Electrolyte and Acid-Base Balance

Section IV: Glomerular Diseases

87

! UA ! Labs to exclude autoimmune renal disease and CKD ! Family history is especially important ! Try to check urine from both parents for hematuria ! Renal biopsy may help in atypical or unusual cases Histopathology ! LM and IF normal ! Normal Type IV collagen immunostaining ! Diffuse thinning of basement membrane on EM

• Need to compare to normals of same age and gender Differential diagnosis ! Alport syndrome

• Alport has irregular thinning of GBM, while TBMD has even, diffuse thinning throughout GBM • Alport has patchy, pale or absent staining for type IV collagen, while TBMD has normal staining

Pathogenesis/pathophysiology ! Unknown ! In contrast to Alport syndrome, patients with TBMD appear to have completely normal type IV collagen (despite

mutations in COL4A3 or COL4A4) Natural history and epidemiology ! At least 1% of population has it, possibly as high as 9% ! Generally excellent prognosis, with normal renal function and no progression

• Very few patients develop hypertension, proteinuria or CKD • In such cases, consider alternate diagnosis such as Alport syndrome

Treatment ! No treatment necessary Genetic transmission Cellular/molecular defect Genetic counseling ! Autosomal dominant [AD] ! Many different mutations described in COL4A3 and COL4A4 genes ! Patients may be heterozygous “carriers” of [ar] Alport syndrome

• Note that the same genes can cause an [ar] and an [AD] disease ! Persons with two mutant alleles (homozygotes) have [ar] Alport syndrome ! Persons with one mutant allele (heterozygotes) have TBMD which behaves as an [AD] trait

! Family history may be absent because of new mutations, or relatives are not aware they have microscopic hematuria

! See chapter 100, Review of basic Mendelian genetics to review counseling on inheritance of [AD] diseases

Fast Facts Alport Syndrome vs. Thin Basement Membrane Disease

Mutant genes COL4A3, 4, or 5 COL4A3 or 4 Transmission AD, ar or xl AD or sporadic Complications Proteinuria, HTN and CKD common Proteinuria, HTN and CKD very rare Prognosis Progressive; high rate of ESRD Not progressive; favorable prognosis Associations Hearing loss, anterior lenticonus, retinopathy No extrarenal findings Histology Irregular thinning of GBM on EM Diffuse, even thinning of GBM on EM Type IV collagen Abnormal type IV collagen Normal type IV collagen

Page 17: Roberts Review -Release version - FINAL - 050211pednephboardreview.com/Welcome_files/Roberts Review... · I. Fetal and Neonatal Nephrology 1 II. Fluid, Electrolyte and Acid-Base Balance

Section IV: Glomerular Diseases

93

! ~ 40% have just microscopic hematuria • Some may have mild proteinuria • Often incidentally discovered

! < 10% have severe findings such as nephrotic syndrome or RPGN Laboratory evaluation ! Definitive diagnosis is only by biopsy ! Elevated levels of IgA or IgA-fibronectin complexes are of only limited help

• Only ~1/3 of patients have elevated IgA levels ! And some patients with very high IgA levels don’t have IgAN

• Still others with IgAN have normal or low IgA levels • So these tests are neither sensitive nor specific

Histopathology ! LM: Mesangial proliferation and matrix expansion ! IF: IgA deposited in mesangium

• May also show IgG, fibrin, C3, and properdin ! EM: Electron-dense deposits in mesangium (outside mesangial cells in mesangial spaces)

• May also be in subendothelial and subepithelial spaces ! Renal pathology is identical to HSP, but HSP is a systemic vasculitis with multiorgan involvement (see chapter 53,

Henoch-Schönlein purpura), while IgAN is limited to kidney and is not a vasculitis Differential diagnosis ! Clinical presentation: Differential diagnosis for gross and/or microscopic hematuria is vast

• Note gross hematuria during respiratory infections with persistent intercurrent microscopic hematuria also occurs with thin basement membrane disease

! Histologic lesion • HSP • IgAN sometimes coexists with minimal change disease • Glomerular IgA deposits are seen in cirrhosis, celiac disease, HIV infection, and even healthy people, but

usually without renal disease Pathogenesis/pathophysiology ! Polymeric form of IgA1 (pIgA1) is main mediator ! Exactly how and why these complexes accumulate in mesangium remains a mystery

• Some disorder of synthesis and/or clearance proposed • Aberrant O-glycosylation of IgA1 is a leading theory

Natural history and epidemiology ! Most common cause of glomerulonephritis ! Not benign; newer data suggest renal disease progresses very slowly

• Patients may develop HTN or decreased renal function over a period of years • After 20 years, 25% of patients need dialysis • However, some cases remit spontaneously

! Bad prognostic signs are same as for other glomerular diseases, e.g. hypertension, tubulointerstitial involvement, high grade proteinuria, etc.

! Recurrence after transplant • Most patients have histologic recurrence, but may not be clinically significant • 15% have significant graft dysfunction • 7% lose graft

Treatment ! Good evidence is lacking and treatment based mainly on opinion ! Possibly no treatment if mild disease (e.g. little or no proteinuria) ! ACE/ARB if hypertension or proteinuria ! Statin drug for hypercholesterolemia ! Fish oil

• Rich in long-chain omega-3 fatty acids eicosapentaenoic acid (EPA) and docosahexaenoic acid (DHA)

Page 18: Roberts Review -Release version - FINAL - 050211pednephboardreview.com/Welcome_files/Roberts Review... · I. Fetal and Neonatal Nephrology 1 II. Fluid, Electrolyte and Acid-Base Balance

Robert’s Review of Pediatric Nephrology by Robert Samuel Gillespie

126

• Manage electrolytes and acid/base status as indicated; watch for dilutional hyponatremia • Limit fluid volume, if indicated (see chapter 13, Edema and ascites) • Optimize nutrition with enteral or parenteral nourishment • Newborns with AKI are very likely to require renal replacement therapy, especially if oliogoanuric

! Medications may comprise a relatively high volume of fluid intake ! Adequate nutrition requires a high volume of fluid and high metabolic load ! See appropriate chapters in section X, Renal replacement therapy

Additional Reading Andreoli SP. Acute renal failure in the newborn. Semin Perinatol 2004;28:112-123. PMID 15200250. Chua AN, Sarwal MM. Acute renal failure management in the neonate. NeoReviews 2005;6:e369-e376. No PMID.

84. Volume depletion

Pathophysiology ! Kidneys release local mediators to increase RBF and perfusion pressure

• Afferent arteriole relaxes (prostaglandins) • Efferent arteriole constricts (angiotensin II)

! With normal tubular function, kidneys appropriately retain salt and water, creating highly concentrated urine with low urine Na and FENa < 1%

Common causes ! True intravascular volume depletion

• Vomiting/diarrhea • Hemorrhage • Diabetes insipidus • Burns • Third-spacing: Sepsis, nephrotic syndrome

! Diminished renal perfusion (functionally similar to volume depletion) • Heart failure • Hepatorenal syndrome • Renal artery stenosis

85. Acute tubular necrosis

Clinical manifestations and risk factors ! Clinical presentation: Acute renal failure

• Diagnosis of ATN often made based on history, clinical findings, and absence of other causes of ARF ! Predisposing factors

• Circulatory impairment (e.g., volume depletion, hypotension, cardiac arrest, cardiac bypass, renal artery thrombosis)

• Hypoxia (e.g., respiratory failure, near-drowning, birth asphyxia) • Nephrotoxins (e.g., drugs, heavy metals, ethylene glycol) • Multiple factors may coexist, e.g., in sepsis

Pathophysiology and natural history ! Ischemia results in depletion of cellular ATP (note kidney has very high oxygen requirement, with S3 segment of

proximal tubule and medullary thick ascending limb being most sensitive to hypoxia) ! Nephrotoxic agents cause direct tubular damage ! Loss of polarity

• Cytoskeleton damage disrupts polarity, resulting in dysfunction • Integrins (cell adhesion molecules) move to incorrect locations resulting in viable cells sloughing off

basement membrane ! Tubular obstruction: Large casts comprised of sloughed epithelial cells, Tamm-Horsfall protein and debris can

obstruct tubular lumen ! Endothelial cell injury: Multiple factors result in damage to endothelium; ultimately inhibits blood flow to renal

cells ! Backleak: Weakened cell adhesion allows glomerular filtrate to back-leak into interstitium, leading to

artifactually lower measured GFR (e.g., serum creatinine) ! Inflammatory mediators: Experimental data suggest leukocyte infiltration, cytokines, complement activation

and reactive oxygen species also play significant role in ATN ! Phospholipase A2 activation: Breaks down membrane phospholipids

Page 19: Roberts Review -Release version - FINAL - 050211pednephboardreview.com/Welcome_files/Roberts Review... · I. Fetal and Neonatal Nephrology 1 II. Fluid, Electrolyte and Acid-Base Balance

Robert’s Review of Pediatric Nephrology by Robert Samuel Gillespie

154

Figure 108.2. Illustration showing grossly “spongy” appearance due to multiple cysts in the collecting ducts. Courtesy of National Institute of Diabetes and Digestive and Kidney Diseases, National Institutes of Health.

Figure 108.3. IV pyelogram in patient with MSK shows characteristic “brush” shapes. Courtesy of Chandra Shekhar Biyani, MD.

Fast Facts

Medullary sponge kidney: Developmental anomaly; Ectatic/cystic changes of all papillary collecting ducts; diffuse cysts, various sizes, in medulla; cortex normal. Imaging: “Paintbrush” or “bouquet of flowers” on IVP; echogenic corticomedullary junction or medullary nephrocalcinosis on US. Ca stones common.

Additional Reading Gambaro G, Feltrin GP, Lupo A, et al. Medullary sponge kidney (Lenarduzzi–Cacchi–Ricci disease): A Padua

Medical School discovery in the 1930s. Kidney Int 2006;69:663–670. PMID 16395272. Exhaustive review and history of MSK.

Rommel D, Pirson Y. Medullary sponge kidney--part of a congenital syndrome. Nephrol Dial Transplant 2001;16:634-6. PMID 11239047. Case report of a syndromic patient, but open access article with good example of IV pyelogram appearance.

109. Tuberous sclerosis

! See also chapter 158, Angiomyolipoma Renal cysts may be presenting signs of tuberous sclerosis ! Cysts and/or angiomyolipomas occur in 70-80% of patients

• When they occur together, are virtually diagnostic for TS ! Cysts can be in medulla and/or cortex (glomerular cysts) Cysts are not related to angiomyolipomas ! Cysts are just cysts – a closed cavity lined with epithelium ! Angiomyolipomas are solid tumors - of blood vessel, smooth muscle, and adipose tissue (think about the name:

angio – myo – lip – oma) – benign, but have a small risk of malignant transformation Histopathology ! Cysts in TS have a unique hyperplastic tall columnar epithelium that may extend into or even obscure the cyst

lumen ! Looks like proximal tubule, but they can arise from anywhere along the nephron ! Distinctly different from the plain cuboidal epithelial lining seen in other cystic renal diseases

Page 20: Roberts Review -Release version - FINAL - 050211pednephboardreview.com/Welcome_files/Roberts Review... · I. Fetal and Neonatal Nephrology 1 II. Fluid, Electrolyte and Acid-Base Balance

Robert’s Review of Pediatric Nephrology by Robert Samuel Gillespie

166

Differential diagnosis: Hypophosphatemia with high serum calcium ! In this case P is usually low because the high Ca binds to and precipitates P ! Malignancy ! Primary or tertiary hyperparathyroidism ! Excess vitamin D ! Immobilization ! Milk-alkali syndrome

• Largely historical disease. Before H2 blockers and proton pump inhibitors existed, the only medical treatment for acid reflux and peptic ulcer was huge amounts of antacids (calcium carbonate or magnesium or aluminum hydroxide) and milk. These patients often developed a profound alkalosis, hypophosphatemia and hypercalcemia, leading to nephrocalcinosis and renal failure

125. Introduction to renal tubular acidosis [supplemental]

Table 124.1: Comparison of types of renal tubular acidosis

Type 1 Dista1

Type 2 ProXimal

Type 4 ALDOsterone resistant or deficient

Pathophysiology Can’t excrete H+ ions in distal tubule (decreased H+ ATPase or back-leak) (this also reduces HCO3-reabsorption)

HCO3- wasting in proximal tubule Impaired excretion of H+ and K+ in distal tubule – defective NH4+ production

Serum HCO3- Low (< 10 mEq/L) Low to normal (10-20 mEq/L) Lowish normal (18-22 mEq/L) Metabolic acidosis

Severe Patients are in acid-base balance, but at a lower set point than normal

Hyperkalemia is key feature; acidosis is secondary & usually mild

Serum anion gap Normal Normal Normal Serum Cl- High High High Serum K+ Low Low to normal High Urine pH Alkaline (unable to acidify urine) Acidic (because distal acidifica-

tion is intact) Acidic

Urine net charge Positive Negative Negative FEHCO3- > 15% < 3% Hypercalciuria/ nephrocalcinosis

Yes No No

Bone disease Yes, due to consumption of bone buffers and hypercalciuria

!Yes, if P wasting occurs. Less common and less severe than type 1. Bone buffers aren’t needed because patients still have acid-base balance

No

Other findings !Urine Na > 25 mEq/L !Polyuria !Sensorineural hearing loss in [ar] form only

!Often transient; often with other proximal tubule dysfunction even if not full Fanconi syndrome !High FEHCO3 during NaHCO3 loading

Causes !Primary !Drugs – Amphotericin, lithium, ifosfamide !Abuse – toluene !Diseases – Medullary sponge kidney !Inherited – [AD] or [ar] !Autoimmune disease

!Primary !Drugs – gentamicin, outdated tetracycline, ifosfamide !Any disease associated with Fanconi syndrome

!Any low-aldosterone state !Congenital adrenal hyperplasia !Addison’s disease !Pseudohypoaldosteronism !Adrenal insufficiency !Obstructive uropathy !Multicystic renal dysplasia !ACE inhibitors

Treatment !Base replacement – modest dose of K citrate

!Base replacement – large doses !P replacement !Vitamin D

!Base replacement !Mineralocorticoid (Fludrocortisone) !Possibly diuretics if hyperkalemic

Page 21: Roberts Review -Release version - FINAL - 050211pednephboardreview.com/Welcome_files/Roberts Review... · I. Fetal and Neonatal Nephrology 1 II. Fluid, Electrolyte and Acid-Base Balance

Robert’s Review of Pediatric Nephrology by Robert Samuel Gillespie

174

! DDAVP is an analog of ADH (or vasopressin) which has a longer half-life and none of the pressor effects of native ADH (or vasopressin)

! Chlorpropamide is a sulfonylurea oral hypoglemic agent that also enhances the response to ADH. Newer oral hypoglycemic drugs do not have this effect. Obviously, hypoglycemia is a possible side effect

! Diuretics cause mild volume depletion, which increases proximal sodium and water reabsorption, thereby diminishing water delivery farther down the nephron to collecting tubules, where ADH (or lack therof) exerts its effect. Paradoxically, this reduces the urine output

! The mechanism of NSAIDs in DI is not fully understood, but evidence suggests that some prostaglandins have anti-ADH effects, so inhibiting prostaglandin synthesis negates these effects and allows ADH to work better

Laboratory evaluation (of polyuria and diabetes insipidus) ! Start by checking serum sodium, urine and serum osmolality ! Water deprivation test

• Protocols vary in details but all have same basic principle • Don’t allow patient access to any fluids • Check serum Na and osmolality, urine osmolality, urine specific gravity, patient weight, vital signs and urine

output every 30-60 minutes • Basically, if patient has DI, he will start to get dehydrated, with weight loss and increase in serum sodium and

osmolality, while still making an inappropriately dilute urine ! The trick here is to detect early signs of this and stop the test before the patient crashes

o Stop if the patient shows a normal urine response… ! Specific gravity 1.020 or osmolality 600 mOsmol/kg

o …or if the patient shows signs of dehydration ! Serum Na elevated or serum osmolality > 295 mOsmol/kg ! Weight loss of $ 5% or tachycardia or hypotension

• If you have to stop the test, check ADH level. It will be very high in patients with nephrogenic DI but low in patients with central DI

• If the patient is stable but has a positive test (high serum Na or osmolality), then give a dose of DDAVP and see if urine output decreases and urine osmolality increases ! Good response to DDAVP – central DI (AVP deficiency) ! Little or no response to DDAVP – nephrogenic DI

! The normal response to water deprivation is to have a huge increase in urine osmolality and maintain normal serum Na and osmolality. If this happens, the patient has primary polydipsia

Genetics

Table 132.2: Genes in nephrogenic DI

% Gene Product Inheritance 90% AVPR2 AVP V2 receptor [xl] 9% AQP2 Aquaporin-2 [ar] 1% AQP2 Aquaporin-2 [AD]

! Function: When AVP binds to V2 receptor, it initiates cascade of events, with end result of Aquaporin-2 water

channels being inserted into luminal membrane. These allow water to move back into the cell (be reabsorbed). Hence a mutation in the V2 receptor (AVPR2) or Aquaporin-2 (AQP2) impairs this process and leads to water wasting.

! AVPR2: X-Linked - Typically only males affected; females are carriers. BUT some females affected due to disproportionate Lyonization

Fast Facts

Causes of Nephrogenic DI: AVPR2 (V2 receptor)[xl]; AQP2 (Aquaporin-2)[ar]/[AD]; lithium or obstruction Treatment: Restrict Na & solutes; thiazide ± amiloride; NSAID; possibly DDAVP; avoid NS infusion

133. Syndrome of inappropriate antidiuretic hormone (SIADH) [supplemental]

Pathogenesis/pathophysiology ! Key triad

• Hyponatremia

Page 22: Roberts Review -Release version - FINAL - 050211pednephboardreview.com/Welcome_files/Roberts Review... · I. Fetal and Neonatal Nephrology 1 II. Fluid, Electrolyte and Acid-Base Balance

Section IX: Chronic Kidney Disease

237

• Have well-trained staff measure length/height and weight at every encounter • For infants, use infant board, not marks on exam table paper • For older children, use stadiometer, not wall chart or height arm on scale

! Plot on correct growth chart • Use 0-36 month chart for measurements on supine infants • Use 2-18 year chart for standing measurements • If below 5th percentile, use growth chart that includes 3rd percentile • Special growth charts available for Down syndrome, Turner syndrome and Williams syndrome

! http://www.cdc.gov/growthcharts - Regular growth carts ! http://www.turnersyndrome.org – Turner syndrome ! http://www.growthcharts.com – Down syndrome ! http://www.williams-sydrome.org – Williams syndrome

! Growth velocity • Determine annually, or at minimum over 6 months

! Shorter time periods prone to inaccuracy due to normal variations • Plot on growth chart

! Standard Deviation Score (SDS or Z-score) • Statistical calculation done using software • Number of standard deviations below (or above) mean for patients of same age/gender

Evaluation of growth potential ! Bone age – use left wrist

• Bone age less than chronological age suggests more growth potential • Best to plot growth chart using bone age rather than chronological age

! Mid-parental height (MPH) • Boys: (Father’s height + Mother’s height + 13 cm)/2 • Girls: (Father’s height + Mother’s height - 13 cm)/2 • Estimate of child’s genetic growth potential – normal children usually grow to ±8.5 cm of the MPH

! This range represents the 3rd to 97th percentiles of anticipated adult height ! Duration and severity of renal disease

• Longer duration and higher degree of renal insufficiency predict poorer growth • Children with chronic renal failure in infancy seldom attain normal growth, even with maximal intervention • Best results obtained in prepubertal children and/or those with milder renal disease (predialysis state)

! Sexual maturity • Again, prepubertal patients have best growth potential • Although a growth spurt occurs with puberty, growth plates also begin to close, terminating growth potential

Evaluation of sexual maturation ! Sexual Maturity Rating (SMR), popularly called Tanner staging, is gold standard. Yields two values from 1 to 5:

• Pubic hair • Breast or genital development

! For males, use orchidometer to measure testicular volume ! Refer to any pediatric text for details Importance of sodium supplementation in infants with renal dysplasia or obstructive uropathy ! Salt wasting is very common with renal dysplasia or obstructive nephropathy, as well as some tubular disorders ! Patients on peritoneal dialysis may also develop hyponatremia due to its very effective Na removal ! Hyponatremia and hypochloremia are associated with growth failure

• Exact relationship poorly understood, but treatment clearly improves growth ! Use sodium supplementation aggressively in any patients who require it ! See also chapter 27, Hyponatremia Indications for and complications of growth hormone therapy ! Note: This objective was relocated from chapter 185, Hemodialysis ! Patients with renal failure have normal levels of GH but need higher than normal levels due to apparent peripheral

resistance to GH (analogous to insulin resistance in type 2 diabetes) ! Many children can achieve catch-up growth and normal adult height with GH

Page 23: Roberts Review -Release version - FINAL - 050211pednephboardreview.com/Welcome_files/Roberts Review... · I. Fetal and Neonatal Nephrology 1 II. Fluid, Electrolyte and Acid-Base Balance

Robert’s Review of Pediatric Nephrology by Robert Samuel Gillespie

300

Figure 205.1: Acute rejection showing inflammation (yellow arrow,) interstitial edema and lymphocytic tubulitis (red arrows.) Courtesy of Jane Bell, MD

Figure 205.2. Fulminant acute rejection with tubulitis (red arrow,) hemorrhage and calcium deposition (circled.) The extremely dark blue cells are lymphocytes. Courtesy of Jane Bell, MD

Figure 205.3. C4d staining in peritubular capillaries, diagnostic of humoral rejection. Courtesy of Chris Larsen, MD

Figure 205.4. Acute rejection with tubulitis; lymphocytes (arrows) are infiltrating the structure of the tubules. Courtesy of Yihan Wang, MD

Immune tolerance ! Tolerance is immune non-responsiveness to specific antigens while still responding to all other nonself antigens ! Possible mechanisms in nature

• Clonal deletion: All T cells which recognize a specific antigen are eliminated while still developing in thymus

• Activation-induced cell death: Activated T cells express “death receptors” which induce apoptosis; over-activation of T cells due to recurrent exposure to antigen results in apoptosis

• Passive cell death: Activated T cells die when they do not get enough growth factors ! Possible clinical applications

• Chimerism: Transplant recipient receives stem cells from the donor’s bone marrow that are present within the graft. Persistence of these cells may help induce tolerance. Tolerance may partly explain why some people don’t reject even with poor compliance with antirejection medication ! This has led to efforts to induce tolerance prior to transplant by deliberately giving recipient some of

donor’s hematopoietic cells, often with some immune suppression as well • Costimulatory blocking: Costimulatory ligand or second signal is essential to activate T cells, so substances

which block it will produce anergy (lack of response in an otherwise normal T cell)

Page 24: Roberts Review -Release version - FINAL - 050211pednephboardreview.com/Welcome_files/Roberts Review... · I. Fetal and Neonatal Nephrology 1 II. Fluid, Electrolyte and Acid-Base Balance

Section XIII: Clinical Research, Biostatistics and Education

347

o Design-stage strategies ! Randomly select participants - best ! Use matching of cases to controls ! Restriction – e.g. limit subjects to nonsmokers

o Analysis-stage strategies ! Stratify results – e.g. divide into smokers and nonsmokers ! Use multivariate statistical methods to control for confounder

Differences in study results among sub-populations (effect modification) ! Effect modification occurs when there is an effect overall, but that effect is different in some subgroups

• Due to statistical interaction in which the effect measure depends on another factor • Effect or association is different in different groups (e.g. men or women, young or old) • Example: If you are studying what proportion of children exposed to varicella become hospitalized, the result

might differ between children who did and did not receive varicella vaccine. Vaccination is an effect modifier

Fast Facts

Validity: Extent to which a study or test measures what it is intended to measure Bias/Systematic error: Causes results to differ from true values in consistent and repeatable manner Confounding: A variable is associated with both the risk factor and the outcome of interest Effect modification: There is an effect overall, but it is different in some subgroups

Additional Reading Hartman JM, Forsen JW, Wallace MS, et al. Tutorials in clinical research: part IV: recognizing and controlling bias.

Laryngoscope 2002;112:23-31. PMID 11802034. Excellent article listing 69(!) named types of bias (including unusual ones such as “hot-stuff bias” and “tidying-up bias”) as well as extensive strategies to reduce them.

232. Causation

Association vs. causation ! Association simply means 2 events occur in conjunction with each other ! Causation means that one event makes the other event happen

Table 232.1: Conditions under which associations are observed

Reason for finding association Type of association Actual relationship between variables studied Chance Spurious None Bias (systematic error) Spurious None Confounding Real Both are associated with another factor that causes event of interest Cause and effect Real Causal

Factors that strengthen causal inference ! The Bradford Hill criteria, devised by Sir Austin Bradford Hill, are widely used to evaluate the strength of causal

inference 1. Strength of association: The stronger the association, the more likely it is to be a causal relationship 2. Temporality (temporal sequence): Cause must occur before effect 3. Dose-response (or biological gradient): Incremental change in effect with change in exposure strongly

supports causality (but is not required) 4. Consistency: A causal relationship should be reproducible in different studies and different populations 5. Coherence with other knowledge: The conclusion of causality makes sense in the context of our other

knowledge about the subject 6. Specificity: Ideally, an effect is associated with only one cause, or the cause is the best predictor of the effect 7. Biologic plausibility: A causal relationship should agree with currently accepted scientific principles 8. Experimental evidence: Other experimental evidence supports the relationship 9. Analogy: The relationship is analogous to other known relationships

! None of these prove causality, but the more of these factors you have, the more confident you can feel in inferring a causal relationship

Page 25: Roberts Review -Release version - FINAL - 050211pednephboardreview.com/Welcome_files/Roberts Review... · I. Fetal and Neonatal Nephrology 1 II. Fluid, Electrolyte and Acid-Base Balance

Section XIII: Clinical Research, Biostatistics and Education

359

Part C: Adult education

243. Educational theory

Basic principles of adult learning theory ! Andragogy is the art and science of helping adults learn. It is very different from pedagogy, which deals with

teaching children ! Andragogy pioneer Eduard Lindeman, in his seminal 1926 book The Meaning of Adult Educationa, identified five

key assumptions about adult learners (as summarized by Knowlesb): 1. Adults are motivated to learn as they experience needs and interests that learning will satisfy. 2. Adults’ orientation to learning is life-centered. 3. Experience is the richest source for adult’s learning. (“Experience is the adult learner’s living textbooka”) 4. Adults have a deep need to be self-directing. 5. Individual differences among people increase with age.

! Contrast with pedagogy below helps illustrate differences (note pedagogy is not on content outline)

Table 243.1: Contrasts between andragogy and pedagogy

Characteristic Andragogy Pedagogy [supplemental] Learner Self-directed; learner referred to as “learner”

Responsible for own learning Learners identify what they want to learn

Instructor-directed; learner referred to as “student” Instructor selects content & technique of learning Learners are told what they are required to learn

Learner’s experience

Learners come with extensive and diverse backgrounds of experience, which forms important part of self-identity and contributes to learning of group

Learners have little experience Instructor’s experience is primary resource

Motivation Internal motivation: self-esteem, recognition, improved quality of job/life

External motivation: required by law, parental guidance, competition for grades

Objective of learning

Solve a problem, function more effectively or achieve goal in life (e.g. job advancement, run business more effectively, career change)

Advance to next level of education

Content Problem-centered: Content must be relevant to real-life issues; learners want to know why they need to know material

Subject-centered: Content is relatively fixed and based on subject matter alone, not practical relevance

Evaluation Self-evaluation plays major role

Evaluation by instructor

Attributes of effective learning environment ! Hutchinson’s principles of adult educationc

• Motivation – may be intrinsic or extrinsic • Belonging – make students feel like important part of the group or team, not an intrusion • Self-esteem – praise good work, provide constructive (not destructive) criticism • Relevance – learners must understand relevance of subject matter to their own goals – key to motivation • Role modeling – teacher is a role model, and learners learn from and often emulate their actions and behavior • Physical facilities

! “Creature comforts:” Adequate and comfortable seating, comfortable temperature, adequate lighting, food/beverages

! Minimize visual and audio distractions (e.g. noise from adjacent rooms or ventilation equipment) ! Appropriate, working equipment

! Knowles’ principles of adult learning environment • Respect for personality: Recognizes that adult learners bring knowledge and life experiences, and learning

is a two-way street • Participation in decision making • Freedom of expression: Learners should feel safe from ridicule in voicing their ideas and identifying gaps in

knowledge • Mutual responsibility for defining goals, activities and evaluation